Difference between revisions of "1965 AHSME Problems/Problem 30"
(Added two solutions) |
m (category) |
||
(6 intermediate revisions by the same user not shown) | |||
Line 1: | Line 1: | ||
− | == Problem | + | == Problem == |
Let <math>BC</math> of right triangle <math>ABC</math> be the diameter of a circle intersecting hypotenuse <math>AB</math> in <math>D</math>. | Let <math>BC</math> of right triangle <math>ABC</math> be the diameter of a circle intersecting hypotenuse <math>AB</math> in <math>D</math>. | ||
Line 8: | Line 8: | ||
\textbf{(C) }\ DF = FA \qquad | \textbf{(C) }\ DF = FA \qquad | ||
\textbf{(D) }\ \angle A = \angle BCD \qquad | \textbf{(D) }\ \angle A = \angle BCD \qquad | ||
− | \textbf{(E) }\ \angle CFD = 2\angle A </math> | + | \textbf{(E) }\ \angle CFD = 2\angle A </math> |
+ | == Solution 1 == | ||
+ | <asy> | ||
+ | |||
+ | path circ, hyp; | ||
+ | |||
+ | draw((0,16)--(0,0)--(8,0)--(0,16)); | ||
+ | dot((0,16)); | ||
+ | label("A", (0,16), NW); | ||
+ | dot((0,0)); | ||
+ | label("C", (0,0), SW); | ||
+ | dot((8,0)); | ||
+ | label("B", (8,0), SE); | ||
+ | |||
+ | dot((0,8)); | ||
+ | label("F",(0,8),W); | ||
+ | |||
+ | markscalefactor=0.1; | ||
+ | draw(rightanglemark((0,16),(0,0),(8,0))); | ||
+ | |||
+ | circ=circle((4,0),4); | ||
+ | draw(circ); | ||
+ | hyp=(0,16)--(8,0); | ||
+ | pair [] x=intersectionpoints(circ,hyp); | ||
+ | dot(x[1]); | ||
+ | label("D",x[1],NE); | ||
+ | draw(x[1]--(0,8)); | ||
+ | draw(x[1]--(0,0)); | ||
+ | |||
+ | </asy> | ||
− | |||
We will prove every result except for <math>\fbox{B}</math>. | We will prove every result except for <math>\fbox{B}</math>. | ||
− | By Thales' Theorem, <math>\angle CDB=90^\circ </math> and so <math>\angle CDA= 90^\circ </math>. <math>FC</math> and <math>FD</math> are both tangents to the same circle, and hence equal. Let <math>\angle CFD=\alpha</math>. Then <math>\angle FDC = \frac{180^\circ - \alpha}{2}</math>, and so <math>\angle FDA = \frac{\alpha}{2}</math>. We also have <math>\angle AFD = 180^\circ - \alpha</math>, which implies <math>\angle FAD=\frac{\alpha}{2}</math>. This means that <math>CF=DF=FA</math>, so <math>DF</math> indeed bisects <math>CA</math>. We also know that <math>\angle BCD=90-\frac{180^\circ - \alpha}{2}=\frac{\alpha}{2}</math>, hence <math>\angle A = \angle BCD</math>. And <math>\angle CFD=2\angle A</math> as <math>\alpha = \frac{\alpha}{2}\times 2</math>. | + | By [[Thales' Theorem]], <math>\angle CDB=90^\circ </math> and so <math>\angle CDA= 90^\circ </math>. <math>FC</math> and <math>FD</math> are both tangents to the same circle, and hence equal. Let <math>\angle CFD=\alpha</math>. Then <math>\angle FDC = \frac{180^\circ - \alpha}{2}</math>, and so <math>\angle FDA = \frac{\alpha}{2}</math>. We also have <math>\angle AFD = 180^\circ - \alpha</math>, which implies <math>\angle FAD=\frac{\alpha}{2}</math>. This means that <math>CF=DF=FA</math>, so <math>DF</math> indeed bisects <math>CA</math>. We also know that <math>\angle BCD=90-\frac{180^\circ - \alpha}{2}=\frac{\alpha}{2}</math>, hence <math>\angle A = \angle BCD</math>. And <math>\angle CFD=2\angle A</math> as <math>\alpha = \frac{\alpha}{2}\times 2</math>. |
Since all of the results except for <math>B</math> are true, our answer is <math>\fbox{B}</math>. | Since all of the results except for <math>B</math> are true, our answer is <math>\fbox{B}</math>. | ||
Line 21: | Line 49: | ||
== Solution 2 == | == Solution 2 == | ||
It's easy to verify that <math>\angle CDA</math> always equals <math>90^\circ</math>. Since <math>\angle CDF</math> changes depending on the sidelengths of the triangle, we cannot be certain that <math>\angle CDF=45^\circ</math>. Hence our answer is <math>\fbox{B}</math>. | It's easy to verify that <math>\angle CDA</math> always equals <math>90^\circ</math>. Since <math>\angle CDF</math> changes depending on the sidelengths of the triangle, we cannot be certain that <math>\angle CDF=45^\circ</math>. Hence our answer is <math>\fbox{B}</math>. | ||
+ | |||
+ | == See Also == | ||
+ | {{AHSME 40p box|year=1965|num-b=29|num-a=31}} | ||
+ | {{MAA Notice}} | ||
+ | [[Category:Introductory Geometry Problems]] |
Latest revision as of 08:29, 19 July 2024
Contents
Problem
Let of right triangle be the diameter of a circle intersecting hypotenuse in . At a tangent is drawn cutting leg in . This information is not sufficient to prove that
Solution 1
We will prove every result except for .
By Thales' Theorem, and so . and are both tangents to the same circle, and hence equal. Let . Then , and so . We also have , which implies . This means that , so indeed bisects . We also know that , hence . And as .
Since all of the results except for are true, our answer is .
Solution 2
It's easy to verify that always equals . Since changes depending on the sidelengths of the triangle, we cannot be certain that . Hence our answer is .
See Also
1965 AHSC (Problems • Answer Key • Resources) | ||
Preceded by Problem 29 |
Followed by Problem 31 | |
1 • 2 • 3 • 4 • 5 • 6 • 7 • 8 • 9 • 10 • 11 • 12 • 13 • 14 • 15 • 16 • 17 • 18 • 19 • 20 • 21 • 22 • 23 • 24 • 25 • 26 • 27 • 28 • 29 • 30 • 31 • 32 • 33 • 34 • 35 • 36 • 37 • 38 • 39 • 40 | ||
All AHSME Problems and Solutions |
The problems on this page are copyrighted by the Mathematical Association of America's American Mathematics Competitions.